Đến nội dung

NTA1907 nội dung

Có 1000 mục bởi NTA1907 (Tìm giới hạn từ 30-04-2020)



Sắp theo                Sắp xếp  

#623732 Topic về Bất đẳng thức, cực trị THCS

Đã gửi bởi NTA1907 on 30-03-2016 - 21:33 trong Bất đẳng thức và cực trị

Cho a,b,c>0.t/m a+b+c=3

Chứng minh:

 

$\frac{a+1}{b^{2}+1}+\frac{b+1}{c^{2}+1}+\frac{c+1}{a^{2}+1}\geq 3$ 

                                 ( hsg tỉnh Nghệ An 2016)

Áp dụng kĩ thuật AM-GM ngược ta có:

$a-\frac{a+1}{b^{2}+1}=\frac{ab^{2}-1}{b^{2}+1}\leq \frac{ab^{2}-1}{2b}=\frac{ab}{2}-\frac{1}{2b}$

$\Leftrightarrow \frac{a+1}{b^{2}+1}\geq a-\frac{ab}{2}+\frac{1}{2b}$

$\Rightarrow \sum \frac{a+1}{b^{2}+1}\geq \sum a-\sum \frac{ab}{2}+\sum \frac{1}{2b}\geq 3-\frac{(a+b+c)^{2}}{6}+\frac{1}{2}.\frac{9}{a+b+c}=3$

Dấu = xảy ra$\Leftrightarrow a=b=c=1$




#605914 Topic về Bất đẳng thức, cực trị THCS

Đã gửi bởi NTA1907 on 29-12-2015 - 12:58 trong Bất đẳng thức và cực trị

Cho a,b,c là các số thực dương thỏa mãn $(a+b)(b+c)(c+a)=1$. Chứng minh rằng:  $ab+bc+ca\leq \frac{3}{4}$

Ta có:

$(a+b)(b+c)(c+a)=(a+b+c)(ab+bc+ca)-abc$

Mà $(a+b+c)(ab+bc+ca)\geq 3\sqrt[3]{abc}.3\sqrt[3]{(abc)^{2}}=9abc$

$\Rightarrow 1\geq (a+b+c)(ab+bc+ca)-\frac{1}{9}(a+b+c)(ab+bc+ca)$

$\Leftrightarrow \frac{81}{64}\geq (a+b+c)^{2}(ab+bc+ca)^{2}\geq 3(ab+bc+ca)^{3}$

$\Rightarrow ab+bc+ca\leq \frac{3}{4}$




#607945 Topic về Bất đẳng thức, cực trị THCS

Đã gửi bởi NTA1907 on 08-01-2016 - 13:16 trong Bất đẳng thức và cực trị

Cho a,b,c là các số dương.Tìm MAX của:

$\frac{\sqrt{ab}}{a+b+2c}+\frac{\sqrt{bc}}{b+c+2a}+\frac{\sqrt{ca}}{c+a+2b}$

Khi thay $(\sqrt{a},\sqrt{b},\sqrt{c})\rightarrow (a,b,c)$ thì bài toán trên tương tự bài toán sau:

918a28593aab437fc1f9a67bab5e2feaf2db6d46.jpg

d5389607c001daec91cb4436a4050bc38ebc3df9.jpg




#608084 Topic về Bất đẳng thức, cực trị THCS

Đã gửi bởi NTA1907 on 08-01-2016 - 22:16 trong Bất đẳng thức và cực trị

Biết $\sum x^{2}=3$

Chứng minh $\sum \frac{xy}{z}\geq 3$

đây




#607734 Topic về Bất đẳng thức, cực trị THCS

Đã gửi bởi NTA1907 on 07-01-2016 - 13:09 trong Bất đẳng thức và cực trị

ai giải bài này đi

Ta có:
$a^{2}+2b+3=(a^{2}+1)+2b+2\geq 2(a+b+1)$
$\Rightarrow \sum \frac{a}{a^{2}+2b+3}\leq \sum \frac{a}{2(a+b+1)}\leq \frac{1}{2}$
Ta cm: $\sum \frac{a}{a+b+1}\leq 1$
$\Leftrightarrow \sum (\frac{a}{a+b+1}+\frac{b+1}{a+b+1})\leq 1+\sum \frac{b+1}{a+b+1}$
$\Leftrightarrow \sum \frac{b+1}{a+b+1}\geq 2$
Ta có:
$\sum \frac{b+1}{a+b+1}=\sum \frac{(b+1)^{2}}{(b+1)(a+b+1)}\geq \frac{(a+b+c+3)^{2}}{\sum (b+1)(a+b+1)}=2$(đpcm)
Dấu = xảy ra$\Leftrightarrow a=b=c=1$



#632975 Topic về Bất đẳng thức, cực trị THCS

Đã gửi bởi NTA1907 on 13-05-2016 - 22:35 trong Bất đẳng thức và cực trị

Chứng minh rằng $\frac{a+b}{\sqrt{a(3a+b)}+\sqrt{b(3b+a)}}\geq\frac{1}{2}$

Ai giúp mình với mình đang cần lắm

Mình cảm ơn trước nhé

Áp dụng Cauchy-Schwarz ta có:

$\frac{a+b}{\sqrt{a(3a+b)}+\sqrt{b(3b+a)}}\geq \frac{a+b}{\sqrt{(a+b)(3a+b+3b+a)}}=\frac{1}{2}$




#624688 Inequalities From 2016 Mathematical Olympiads

Đã gửi bởi NTA1907 on 03-04-2016 - 22:36 trong Bất đẳng thức - Cực trị

Bài 8 (Selection Of Kiev To UMO). Với $a,\,b,\,c$ là ba số thực dương thỏa mãn $a+b+c=3.$ Chứng minh rằng

\[\frac{a^2}{a+b^2}+\frac{b^2}{b+c^2}+\frac{c^2}{c+a^2} \geqslant \frac{3}{2}.\]

Áp dụng AM-GM ta có:

$a-\frac{a^{2}}{a+b^{2}}=\frac{ab^{2}}{a+b^{2}}\leq \frac{ab^{2}}{2b\sqrt{a}}=\frac{b\sqrt{a}}{2}$

Tương tự cộng lại ta được:

$\sum \frac{a}{a+b^{2}}\geq 3-\frac{1}{2}(b\sqrt{a}+c\sqrt{b}+a\sqrt{c})\geq 3-\frac{1}{2}.\sqrt{(a+b+c)(ab+bc+ca)}\geq 3-\frac{1}{2}.\sqrt{(a+b+c).\frac{(a+b+c)^{2}}{3}}=3-\frac{3}{2}=\frac{3}{2}$

Dấu = xảy ra$\Leftrightarrow a=b=c=1$




#625270 Inequalities From 2016 Mathematical Olympiads

Đã gửi bởi NTA1907 on 05-04-2016 - 22:41 trong Bất đẳng thức - Cực trị

Bài 3 (Korea Winter Program Practice Test). Cho ba số thực không âm $x,\,y,\,z$ thỏa mãn

\[(x+y-1)^2+(y+z-1)^2+(z+x-1)^2=27.\]
Tìm giá trị lớn nhất của biểu thức $x^4+y^4+z^4.$

Bài này đề gốc là tìm cả min cả max chứ anh?

302f2d34349b033bf9410dd512ce36d3d439bdfe.jpg




#632382 Inequalities From 2016 Mathematical Olympiads

Đã gửi bởi NTA1907 on 10-05-2016 - 22:31 trong Bất đẳng thức - Cực trị

Bài 32. (Kyiv Mathematical Festival)

1. Cho ba số thực không âm $a,b,c$ thỏa mãn điều kiện $ab+bc+ca=2.$ Chứng minh rằng \[\frac{ab}{c+1}+\frac{bc}{a+1}+\frac{ca}{b+1}+2(a+b+c) \geqslant 6.\]

2. Cho ba số thực không âm $a,b,c$ thỏa mãn điều kiện $ab+bc+ca=3.$ Chứng minh rằng \[\frac{ab}{c+1}+\frac{bc}{a+1}+\frac{ca}{b+1} \geqslant \frac{3}{2}.\]

2. Áp dụng Cauchy-Schwarz ta có:

$\sum \frac{(ab)^{2}}{abc+ab}\geq \frac{(ab+bc+ca)^{2}}{3abc+ab+bc+ca}=\frac{3}{abc+1}\geq \frac{3}{2}$(vì $abc\leq 1$ theo AM-GM)

Dấu = xảy ra khi $a=b=c=1$

1. Tương tự câu 2 ta chứng minh được:

$\sum \frac{ab}{c+1}\geq 6-2\sqrt{6}$

Mà $2(a+b+c)\geq 2\sqrt{3(ab+bc+ca)}=2\sqrt{6}$

Cộng 2 bất đẳng thức trên ta có đpcm




#624680 Inequalities From 2016 Mathematical Olympiads

Đã gửi bởi NTA1907 on 03-04-2016 - 22:24 trong Bất đẳng thức - Cực trị

Bài 6 (Hong Kong TST). Cho ba số thực dương $a,\,b,\,c$ thỏa mãn $abc=1.$ Tìm giá trị nhỏ nhất của

$$\frac{a^3+8}{a^3(b+c)}+\frac{b^3+8}{b^3(a+c)}+\frac{c^3+8}{c^3(b+a)}.$$

 

Không biết giải bài ở đây có vi phạm không anh nhỉ :( 
$VT=\sum \frac{a^3+1+1+6}{a^3(b+c)} \ge \sum \frac{3a+6}{a^3(b+c)}=\sum \frac{3(a+2)}{a^3(b+c)}$ 
Ta sẽ chứng minh $\sum \frac{3(a+2)}{a^3(b+c)} \ge \frac{27}{2}$ (*)
Chợt nhận thấy bài toán quen thuộc của IMO 1995  
Nếu $abc=1$ thì $\sum \frac{1}{a^3(b+c)} \ge \frac{3}{2}$ 
Áp dụng suy ra $\frac{6}{a^3(b+c)} \ge 9$ 
Lại có $\sum \frac{3}{a^2(b+c)}=\sum \frac{3(bc)^2}{b+c} \ge \frac{3(\sum ab)^2}{2\sum a}$ (1) 
Lại có $(\sum ab)^2 \ge 3.abc(a+b+c)$ nên từ (1) suy ra $\frac{3(\sum ab)^2}{2\sum a} \ge \frac{9}{2}$ 
Cộng lại suy ra (*) được chứng minh 
Vậy giá trị nhỏ nhất là $\frac{27}{2}$ khi và chỉ khi $a=b=c=1$

Cách khác

Ta có:
$\sum \frac{a^{3}+8}{a^{3}(b+c)}=\sum \frac{1}{b+c}+\sum \frac{8}{a^{3}(b+c)}\geq \frac{9}{2(a+b+c)}+\sum \frac{8(bc)^{2}}{a(b+c)}\geq \frac{9}{2(a+b+c)}+\frac{8(bc+ca+ab)^{2}}{2(ab+bc+ca)}=\frac{9}{2(a+b+c)}+(ab+bc+ca)+3(ab+bc+ca)\geq \frac{9}{2(a+b+c)}+\sqrt{3abc(a+b+c)}+3.3\sqrt[3]{(abc)^{2}}=\frac{9}{2(a+b+c)}+\frac{\sqrt{3(a+b+c)}}{2}+\frac{\sqrt{3(a+b+c)}}{2}+9\geq 3\sqrt[3]{\frac{27}{8}}+9=\frac{27}{2}$
Dấu = xảy ra khi và chỉ khi $a=b=c=1$



#606298 $\boxed{\text{Chuyên Đề}}$ Phương trình vô tỉ - Hệ phương...

Đã gửi bởi NTA1907 on 31-12-2015 - 14:26 trong Phương trình, hệ phương trình và bất phương trình

8/     $\sqrt{3x^{2}+6x+7}+\sqrt{5x^{2}+10x+14}=4-2x-x^{2}$

$VT=\sqrt{3(x+1)^{2}+4}+\sqrt{5(x+1)^{2}+9}\geq \sqrt{4}+\sqrt{9}=5$

$VP=5-(x+1)^{2}\leq 5$

$VT=VP\Leftrightarrow x=-1$




#635092 Topic về phương trình và hệ phương trình

Đã gửi bởi NTA1907 on 23-05-2016 - 22:25 trong Phương trình - hệ phương trình - bất phương trình

Bài 447:Các bài tập có cùng dạng:

1)$x^2=4+2\sqrt{2x+4}$

2)$5x^2+1=2\sqrt{\frac{2x}{5}+\frac{1}{5}}$

3)$4x^2+4x+1=2\sqrt{4x+2}$

4)$49x^2-65x+17=3\sqrt{2x+1}$

5)$75x^2-79x+28=2\sqrt{3x-4}$

1, ĐK: $x\geq -2$

Đặt $\sqrt{2x+4}=t$

Ta có hệ: $\left\{\begin{matrix} &x^{2}=4+2t \\ &t^{2}=4+2x \end{matrix}\right.$

Dễ rồi...

Các bài còn lại đều tương tự, dạng này ở topic có rất nhiều, bạn có thể tham khảo ở các trang trước




#634687 Topic về phương trình và hệ phương trình

Đã gửi bởi NTA1907 on 22-05-2016 - 13:24 trong Phương trình - hệ phương trình - bất phương trình

Đây là những bài tập chưa có lời giải trong Topic về phương trình và hệ phương trình, mong các bạn sớm hoàn thiện những bài tập này trước khi đăng bài mới để tránh loãng topic

 

Bài 287: $\left\{\begin{matrix} (x+\sqrt{y^{2}+2015})(y+\sqrt{x^{2}+2015})=2015 & & \\ x+y+\sqrt{x+3}=x\sqrt[3]{x+7} & & \end{matrix}\right.$

 

P/s: Những bài có đáp án sẽ được tô màu đỏ.

Đặt $2015=t$

Khi đó pt(1)$\Leftrightarrow (x+\sqrt{y^{2}+t})(y+\sqrt{x^{2}+t})=t$

$\Leftrightarrow xy+x\sqrt{x^{2}+t}+y\sqrt{y^{2}+t}+\sqrt{(x^{2}+t)(y^{2}+t)}=t$

$\Leftrightarrow xy+\sqrt{(x^{2}+t)(y^{2}+t)}-t=-(x\sqrt{x^{2}+t}+y\sqrt{y^{2}+t})$

Bình phương 2 vế ta có:

$x^{2}y^{2}+x^{2}y^{2}+ty^{2}+tx^{2}+t^{2}+t^{2}+2xy\sqrt{(x^{2}+t)(y^{2}+t)}-2xyt-2t\sqrt{(x^{2}+t)(y^{2}+t)}=x^{4}+tx^{2}+y^{4}+ty^{2}+2xy\sqrt{(x^{2}+t)(y^{2}+t)}$

$\Leftrightarrow 2x^{2}y^{2}+2t^{2}-2t\sqrt{(x^{2}+t)(y^{2}+t)}-2xyt=x^{4}+y^{4}$

$\Leftrightarrow (x^{2}-y^{2})^{2}=2\left [ t^{2}-xyt-t\sqrt{(x^{2}+t)(y^{2}+t)} \right ]\geq 0$

$\Rightarrow t^{2}-xyt-t\sqrt{(x^{2}+t)(y^{2}+t)}\geq 0$

$\Leftrightarrow t-xy\geq \sqrt{(x^{2}+t)(y^{2}+t)}$

$\Rightarrow x^{2}y^{2}-2txy+t^{2}\geq x^{2}y^{2}+t(x^{2}+y^{2})+t^{2}$

$\Leftrightarrow t(x+y)^{2}\leq 0$

$\Leftrightarrow x+y=0$

Thay vào pt(2) ta được:

$\sqrt{x+3}=x\sqrt[3]{x+7}$(*)

ĐK: $x\geq 0$

(*)$\Leftrightarrow (\sqrt{x+3}-2)=2x-2+x(\sqrt[3]{x+7}-2)$

$\Leftrightarrow \frac{x-1}{\sqrt{x+3}+2}=2(x-1)+\frac{x(x-1)}{\sqrt[3]{(x+7)^{2}+2\sqrt[3]{x+7}+4}}$

$\Leftrightarrow x=1$ hoặc $\frac{1}{\sqrt{x+3}+2}=2+\frac{x}{\sqrt[3]{(x+7)^{2}+2\sqrt[3]{x+7}+4}}$(**)

Ta dễ dàng chứng minh được pt(**) vô nghiệm với $x\geq 0$

Vậy $(x,y)=(1;-1)$




#643912 Topic về phương trình và hệ phương trình

Đã gửi bởi NTA1907 on 06-07-2016 - 22:33 trong Phương trình - hệ phương trình - bất phương trình

Bài 460: Giải hệ phương trình: $\left\{\begin{matrix} &x\sqrt{12-y}+\sqrt{y(12-x^{2})}=12 \\ &x^{3}-8x-1=2\sqrt{y-2} \end{matrix}\right.$

 

Spoiler




#649568 Topic về phương trình và hệ phương trình

Đã gửi bởi NTA1907 on 14-08-2016 - 11:15 trong Phương trình - hệ phương trình - bất phương trình

Có khá nhiều bài khó trong topic chưa có lời giải. Vì vậy ta sẽ tiếp tục với một bài dễ hơn như sau:

Bài 476: Giải hệ phương trình:

$\left\{\begin{matrix} &(5-x)(1+x^{4}y^{4})=(1+x^{2}y^{2})^{3} \\ &x^{2}y^{2}+x^{2}+x+y^{2}=4 \end{matrix}\right.$




#656043 Topic về phương trình và hệ phương trình

Đã gửi bởi NTA1907 on 29-09-2016 - 21:35 trong Phương trình - hệ phương trình - bất phương trình

Bài 544: $\left\{\begin{matrix} &2(\sqrt{x+1}+1)^{2}=\sqrt[3]{x^{2}+4y+16} \\ &x^{2}+\dfrac{4y}{x}=2(9x-1)\sqrt{2x^{3}-y} \end{matrix}\right.$




#654225 Topic về phương trình và hệ phương trình

Đã gửi bởi NTA1907 on 14-09-2016 - 21:51 trong Phương trình - hệ phương trình - bất phương trình

Bất đẳng thức Cauchy-Schwarz nếu mình không nhớ không nhầm thì là vầy:

$$\left ( ax+by \right )^{2}\leq \left ( a^{2}+b^{2} \right )\left ( x^{2}+y^{2} \right )$$

Nếu vậy thì sao có đoạn này được nhỉ? Mình không rành bất đẳng thức cho lắm nên có gì sai sót mong bỏ qua...

Đây là lỗi của e khi ghi sai đề. Nhưng nếu bài toán như lúc đầu thì ý tưởng của Lawliet hẳn đã tối ưu nhất chưa

Bài 527: $\left\{\begin{matrix} &x\sqrt{1-y^{2}}+y\sqrt{x^{2}-1}=1 \\ &3x^{2}-xy^{2}+4x=1 \end{matrix}\right.$




#633579 Topic về phương trình và hệ phương trình

Đã gửi bởi NTA1907 on 16-05-2016 - 22:25 trong Phương trình - hệ phương trình - bất phương trình

Đây là những bài tập chưa có lời giải trong Topic về phương trình và hệ phương trình, mong các bạn sớm hoàn thiện những bài tập này trước khi đăng bài mới để tránh loãng topic

 

 

Bài 298: $\left\{\begin{matrix} x^3+\frac{1}{3}y=x^2+x-\frac{4}{3}\\y^3+\frac{1}{4}z=y^2+y-\frac{5}{4} \\ z^3+\frac{1}{5x}=z^2+z-\frac{6}{5} \end{matrix}\right.$

 

P/s: Những bài có đáp án sẽ được tô màu đỏ.

Hpt$\Leftrightarrow \left\{\begin{matrix} &(x-1)^{2}(x+1)+\frac{1}{3}(y+1)=0 & \\ &(y-1)^{2}(y+1)+\frac{1}{3}(z+1)=0 & \\ &(z-1)^{2}(z+1)+\frac{1}{3}(x+1)=0 & \end{matrix}\right.$

+) $x=y=z=-1\Rightarrow$ Thoả mãn

+) $x> -1$, từ pt(1)$\Rightarrow y< -1$

$\Rightarrow$ Từ pt(2)$\Rightarrow z> -1$

$\Rightarrow$ Từ pt(3)$\Rightarrow x< -1$(mâu thuẫn)

+) $x< -1$ cũng không thoả mãn

Vậy $(x,y,z)=(-1;-1;-1)$




#615175 Topic về phương trình và hệ phương trình

Đã gửi bởi NTA1907 on 15-02-2016 - 17:05 trong Phương trình - hệ phương trình - bất phương trình

Đây là những bài tập chưa có lời giải trong Topic về phương trình và hệ phương trình, mong các bạn sớm hoàn thiện những bài tập này trước khi đăng bài mới để tránh loãng topic

Bài 181: $(4x-1)\sqrt{x^{2}+1}=2x^{2}+2x+1$

P/s: Những bài có lời giải đã được tô màu đỏ

Bài này ta có 1 cách khá hay  :)

Đặt $y=\sqrt{x^{2}+1}\geq 1\Rightarrow x^{2}=y^{2}-1$

Khi đó pt$\Leftrightarrow (4x-1)y=2(y^{2}-1)+2x+1$

$\Leftrightarrow 2y^{2}-(4x-1)y+2x-1=0$

$\Delta =(4x-1)^{2}-8(2x-1)=16x^{2}-24x+9=(4x-3)^{2}$

$\Rightarrow$ Pt có 2 nghiệm $y=2x-1$(TM) và $y=\frac{1}{2}$(loại)

$\Rightarrow \sqrt{x^{2}+1}=2x-1$

Dễ rồi...




#610752 Topic về phương trình và hệ phương trình

Đã gửi bởi NTA1907 on 24-01-2016 - 16:53 trong Phương trình - hệ phương trình - bất phương trình

Đây là những bài tập chưa có lời giải trong Topic về phương trình và hệ phương trình, mong các bạn sớm hoàn thiện những bài tập này trước khi đăng bài mới để tránh loãng topic

Bài 106: $2(2x-3)(\sqrt[3]{x-1}+\sqrt{x-1})=3x-2$

ĐK: $x\geq 1$

$\Rightarrow VP> 0\Rightarrow 2x-3> 0\Leftrightarrow x> \frac{3}{2}$

Pt$\Leftrightarrow 2(2x-3)\left [ (\sqrt[3]{x-1}-1)+(\sqrt{x-1}-1) \right ]=3x-2-4(2x-3)$

$\Leftrightarrow 2(2x-3)\left [ \frac{x-2}{\sqrt[3]{(x-1)^{2}}+\sqrt[3]{x-1}+1}+\frac{x-2}{\sqrt{x-1}+1} \right ]+5x-10=0$

$\Leftrightarrow 2(2x-3)(x-2)\left [ \frac{1}{\sqrt[3]{(x-1)^{2}}+\sqrt[3]{x-1}+1}+\frac{1}{\sqrt{x-1}+1} \right ]+5(x-2)=0$

$\Leftrightarrow (x-2)(\frac{2(2x-3)}{\sqrt[3]{(x-1)^{2}}+\sqrt[3]{x-1}+1}+\frac{2(2x-3)}{\sqrt{x-1}+1}+5)=0$

$\Rightarrow x=2$(vì phần trong ngoặc luôn dương)




#611710 Topic về phương trình và hệ phương trình

Đã gửi bởi NTA1907 on 29-01-2016 - 22:18 trong Phương trình - hệ phương trình - bất phương trình

đề nghị chủ topic thống kê lại những bài làm rồi và chưa làm đi nào

Bạn có thể xem ở trang 16




#610212 Topic về phương trình và hệ phương trình

Đã gửi bởi NTA1907 on 21-01-2016 - 21:43 trong Phương trình - hệ phương trình - bất phương trình

Bài 85: $\frac{9x^{2}-14x+25}{3x+3+4\sqrt{2x-1}}=\frac{(\sqrt{x-1}-1)(2x-4)}{x}$

Bài 86: $\sqrt{2(4x^{2}-x-6)}-\sqrt{2x-3}=\sqrt{2x^{2}+x-1}$

Bài 87: $\left\{\begin{matrix} &\sqrt{5x^{2}+2xy+2y^{2}}+\sqrt{2x^{2}+2xy+5y^{2}}=3(x+y) \\ &\sqrt{x+2y+1}+2\sqrt[3]{12x+7y+8}=2xy+x+5 \end{matrix}\right.$




#626463 Topic về phương trình và hệ phương trình

Đã gửi bởi NTA1907 on 10-04-2016 - 21:33 trong Phương trình - hệ phương trình - bất phương trình

bài 378: mình giải được rồi (bằng phương pháp liên hợp). Bạn nào có cách giải khác chia sẻ mình nhé.

Nếu bạn đã làm được bài này bằng pp liên hợp thì bạn cứ post lời giải lên để mọi người tham khảo  :)




#608748 Topic về phương trình và hệ phương trình

Đã gửi bởi NTA1907 on 13-01-2016 - 13:15 trong Phương trình - hệ phương trình - bất phương trình

Bài 14: Giải PT: $(x-1)(2\sqrt{x-1}+3\sqrt[3]{x+6})=x+6$

ĐK: $x\geq 1$
Pt$\Leftrightarrow 2(x-1)(\sqrt{x-1}-1)+3(x-1)(\sqrt[3]{x+6}-2)=x+6-2(x-1)-6(x-1)$
$\Leftrightarrow 2(x-1).\frac{x-2}{\sqrt{x-1}+1}+3(x-1).\frac{x-2}{\sqrt[3]{(x+6)^{2}}+2\sqrt[3]{x+6}+4}+7(x-2)=0$
$\Leftrightarrow (x-2)(\frac{2(x-1)}{\sqrt{x-1}+1}+\frac{3(x-1)}{\sqrt[3]{(x+6)^{2}}+2\sqrt[3]{x+6}+4}+7)=0$$
Vì VT của pt cuối luôn dương nên $x=2$(TM)



#624673 Topic về phương trình và hệ phương trình

Đã gửi bởi NTA1907 on 03-04-2016 - 22:04 trong Phương trình - hệ phương trình - bất phương trình

Bài 362: $\begin{cases} & \sqrt{5x^{2}+2xy+2y^{2}}+\sqrt{5y^{2}+2xy+2x^{2}}=3(x+y) \\ & 2x^{2}y+xy^{2}-47x+66=\sqrt[3]9xy+43x+6{} \end{cases}$

Ta có:

$\sqrt{5x^{2}+2xy+2y^{2}}=\sqrt{(2x+y)^{2}+(x-y)^{2}}\geq 2x+y$

Tương tự: $\sqrt{5y^{2}+2xy+2x^{2}}\geq 2y+x$

$\Rightarrow VT\geq VP$

Dấu = xảy ra$\Leftrightarrow x=y$

Thay vào pt(2) là pt bậc 3 nhưng số hơi xấu nên có lẽ phải dùng công thức cardano

P/s: Bài này phải là bài 363 nhé, a sử lại đi